格子定数の可能な値


1

回折格子に関する質問があります。次のようになります。

回折格子があると仮定します。このグレーティングを通して異なる波長の光を照らすと、最大量が異なります。格子が次の条件を満たす必要があるとしましょう:波長光を放つ場合、最大値を取得し、波長光を放つ場合、最大値を取得します。上記の基準が与えられた場合、格子定数のどの値が可能ですか?λ13λ25

格子方程式を使用してこれを解決しようとしました。ここで、3最大値に対してを設定し、最大値に対してを設定してから、角度異なる値を調べます。最大角度として。しかし、この方法を使用して、私はそれを見つけるD > λ 1のための3最大値、およびD > 2 λ 2のための5私には間違ったと思われる最大値を。この答えは間違っていますか?dsina=kλk=1k=25a90d>λ13d>2λ25

すべての助けを本当に感謝します!!


2
私はそれが物理学の問題だと、ここで答えを持っているので、オフトピックとして、この質問を閉じるために投票していますphysics.stackexchange.com/questions/230752/...を
カール・Witthoft
弊社のサイトを使用することにより、あなたは弊社のクッキーポリシーおよびプライバシーポリシーを読み、理解したものとみなされます。
Licensed under cc by-sa 3.0 with attribution required.